UWM 450 Exam 1 chapter 2

Lakukan tugas rumah & ujian kamu dengan baik sekarang menggunakan Quizwiz!

. A portfolio's risk is measured by the weighted average of the standard deviations of the securities in the portfolio. It is this aspect of portfolios that allows investors to combine stocks and thus reduce the riskiness of their portfolios. T/F

FALSE

A stock with a beta equal to −1.0 has zero systematic (or market) risk. T/F

FALSE

A stock's beta is more relevant as a measure of risk to an investor who holds only one stock than to an investor who holds a well-diversified portfolio. T/F

FALSE

A stock's beta measures its diversifiable risk relative to the diversifiable risks of other firms. T/F

FALSE

An individual stock's diversifiable risk, which is measured by its beta, can be lowered by adding more stocks to the portfolio in which the stock is held. T/F

FALSE

Even if the correlation between the returns on two securities is +1.0, if the securities are combined in the correct proportions, the resulting 2-asset portfolio will have less risk than either security held alone. T/F

FALSE

If an investor buys enough stocks, he or she can, through diversification, eliminate all of the market risk inherent in owning stocks, but as a general rule it will not be possible to eliminate all diversifiable risk. T/F

FALSE

If investors become less averse to risk, the slope of the Security Market Line (SML) will increase. T/F

FALSE

If you plotted the returns of a company against those of the market and found that the slope of your line was negative, the CAPM would indicate that the required rate of return on the stock should be less than the risk-free rate for a well-diversified investor, assuming that the observed relationship is expected to continue in the future. T/F

FALSE

Managers should under no conditions take actions that increase their firm's risk relative to the market, regardless of how much those actions would increase the firm's expected rate of return. T/F

FALSE

Portfolio A has but one stock, while Portfolio B consists of all stocks that trade in the market, each held in proportion to its market value. Because of its diversification, Portfolio B will by definition be riskless. T/F

FALSE

The CAPM is a multi-period model that takes account of differences in securities' maturities, and it can be used to determine the required rate of return for any given level of systematic risk. T/F

FALSE

The CAPM is built on historic conditions, although in most cases we use expected future data in applying it. Because betas used in the CAPM are calculated using expected future data, they are not subject to changes in future volatility. This is one of the strengths of the CAPM. T/F

FALSE

The SML relates required returns to firms' systematic (or market) risk. The slope and intercept of this line can be influenced by a manager's actions. T/F

FALSE

The Y-axis intercept of the SML indicates the required return on an individual asset whenever the realized return on an average (b = 1) stock is zero. T/F

FALSE

The distributions of rates of return for Companies AA and BB are given below: State of the Probability of Economy| This State Occurring| AA | BB Boom 0.2 30% −10% Normal 0.6 10% 5% Recession 0.2 −5% 50% We can conclude from the above information that any rational, risk-averse investor would be better off adding Security AA to a well-diversified portfolio over Security BB. a. True b. False

FALSE

The realized return on a stock portfolio is the weighted average of the expected returns on the stocks in the portfolio. T/F

FALSE

The slope of the SML is determined by the value of beta. T/F

FALSE

The tighter the probability distribution of its expected future returns, the greater the risk of a given investment as measured by its standard deviation. T/F

FALSE

Two conditions are used to determine whether or not a stock is in equilibrium: (1) Does the stock's market price equal its intrinsic value as seen by the marginal investor, and (2) does the expected return on the stock as seen by the marginal investor equal this investor's required return? If either of these conditions, but not necessarily both, holds, then the stock is said to be in equilibrium. T/F

FALSE

Under the CAPM, the required rate of return on a firm's common stock is determined only by the firm's market risk. If its market risk is known, and if that risk is expected to remain constant, then analysts have all the information they need to calculate the firm's required rate of return. T/F

FALSE

We would almost always find that the beta of a diversified portfolio is less stable over time than the beta of a single security. T/F

FALSE

We would generally find that the beta of a single security is more stable over time than the beta of a diversified portfolio. T/F

FALSE

Your friend is considering adding one additional stock to a 3-stock portfolio, to form a 4-stock portfolio. She is highly risk averse and has asked for your advice. The three stocks currently held all have b = 1.0, and they are perfectly positively correlated with the market. Potential new Stocks A and B both have expected returns of 15%, are in equilibrium, and are equally correlated with the market, with r = 0.75. However, Stock A's standard deviation of returns is 12% versus 8% for Stock B. Which stock should this investor add to his or her portfolio, or does the choice not matter? a. Stock A. b. Stock B. c. Neither A nor B, as neither has a return sufficient to compensate for risk. d. Add A, since its beta must be lower. e. Either A or B, i.e., the investor should be indifferent between the two.

Stock B.

"Risk aversion" implies that investors require higher expected returns on riskier than on less risky securities. T/F

TRUE

. Any change in its beta is likely to affect the required rate of return on a stock, which implies that a change in beta will likely have an impact on the stock's price, other things held constant. T/F

TRUE

. It is possible for a firm to have a positive beta, even if the correlation between its returns and those of another firm is negative. T/F

TRUE

. One key conclusion of the Capital Asset Pricing Model is that the value of an asset should be measured by considering both the risk and the expected return of the asset, assuming that the asset is held in a well-diversified portfolio. The risk of the asset held in isolation is not relevant under the CAPM. T/F

TRUE

A firm can change its beta through managerial decisions, including capital budgeting and capital structure decisions. T/F

TRUE

According to the Capital Asset Pricing Model, investors are primarily concerned with portfolio risk, not the risks of individual stocks held in isolation. Thus, the relevant risk of a stock is the stock's contribution to the riskiness of a well-diversified portfolio. T/F

TRUE

Assume that two investors each hold a portfolio, and that portfolio is their only asset. Investor A's portfolio has a beta of minus 2.0, while Investor B's portfolio has a beta of plus 2.0. Assuming that the unsystematic risks of the stocks in the two portfolios are the same, then the two investors face the same amount of risk. However, the holders of either portfolio could lower their risks, and by exactly the same amount, by adding some "normal" stocks with beta = 1.0. T/F

TRUE

Bad managerial judgments or unforeseen negative events that happen to a firm are defined as "company-specific," or "unsystematic," events, and their effects on investment risk can in theory be diversified away. T/F

TRUE

Diversification will normally reduce the riskiness of a portfolio of stocks. T/F

TRUE

For a stock to be in equilibrium, two conditions are necessary: (1) The stock's market price must equal its intrinsic value as seen by the marginal investor and (2) the expected return as seen by the marginal investor must equal this investor's required return. T/F

TRUE

If a stock's expected return as seen by the marginal investor exceeds this investor's required return, then the investor will buy the stock until its price has risen enough to bring the expected return down to equal the required return. T/F

TRUE

If a stock's market price exceeds its intrinsic value as seen by the marginal investor, then the investor will sell the stock until its price has fallen down to the level of the investor's estimate of the intrinsic value. T/F

TRUE

If investors are risk averse and hold only one stock, we can conclude that the required rate of return on a stock whose standard deviation is 0.21 will be greater than the required return on a stock whose standard deviation is 0.10. However, if stocks are held in portfolios, it is possible that the required return could be higher on the stock with the low standard T/F

TRUE

If the price of money (e.g., interest rates and equity capital costs) increases due to an increase in anticipated inflation, the risk-free rate will also increase. If there is no change in investors' risk aversion, then the market risk premium (rM − rRF) will remain constant. Also, if there is no change in stocks' betas, then the required rate of return on each stock as measured by the CAPM will increase by the same amount as the increase in expected inflation T/F

TRUE

If the returns of two firms are negatively correlated, then one of them must have a negative beta. T/F

TRUE

In portfolio analysis, we often use ex post (historical) returns and standard deviations, despite the fact that we are really interested in ex ante (future) data. T/F

TRUE

Market risk refers to the tendency of a stock to move with the general stock market. A stock with above-average market risk will tend to be more volatile than an average stock, and its beta will be greater than 1.0. T/F

TRUE

Portfolio A has but one security, while Portfolio B has 100 securities. Because of diversification effects, we would expect Portfolio B to have the lower risk. However, it is possible for Portfolio A to be less risky. T/F

TRUE

Risk-averse investors require higher rates of return on investments whose returns are highly uncertain, and most investors are risk averse. T/F

TRUE

Since the market return represents the expected return on an average stock, the market return reflects a certain amount of risk. As a result, there exists a market risk premium, which is the amount over and above the risk-free rate, that is required to compensate stock investors for assuming an average amount of risk. T/F

TRUE

Someone who is risk averse has a general dislike for risk and a preference for certainty. If risk aversion exists in the market, then investors in general are willing to accept somewhat lower returns on less risky securities. Different investors have different degrees of risk aversion, and the end result is that investors with greater risk aversion tend to hold securities with lower risk (and therefore a lower expected return) than investors who have more tolerance for risk. T/F

TRUE

The Y-axis intercept of the SML represents the required return of a portfolio with a beta of zero, which is the risk-free rate. T/F

TRUE

The slope of the SML is determined by investors' aversion to risk. The greater the average investor's risk aversion, the steeper the SML. T/F

TRUE

Variance is a measure of the variability of returns, and since it involves squaring the deviation of each actual return from the expected return, it is always larger than its square root, its standard deviation. T/F

TRUE

When adding a randomly chosen new stock to an existing portfolio, the higher (or more positive) the degree of correlation between the new stock and stocks already in the portfolio, the less the additional stock will reduce the portfolio's risk. T/F

TRUE

Which of the following statements is CORRECT? a. A portfolio that consists of 40 stocks that are not highly correlated with "the market" will probably be less risky than a portfolio of 40 stocks that are highly correlated with the market, assuming the stocks all have the same standard deviations. b. A two-stock portfolio will always have a lower beta than a one-stock portfolio. c. If portfolios are formed by randomly selecting stocks, a 10-stock portfolio will always have a lower beta than a one-stock portfolio. d. A stock with an above-average standard deviation must also have an above-average beta. e. A two-stock portfolio will always have a lower standard deviation than a one-stock portfolio.

a. A portfolio that consists of 40 stocks that are not highly correlated with "the market" will probably be less risky than a portfolio of 40 stocks that are highly correlated with the market, assuming the stocks all have the same standard deviations.

Which of the following statements is CORRECT? a. Diversifiable risk can be reduced by forming a large portfolio, but normally even highly-diversified portfolios are subject to market (or systematic) risk. b. A large portfolio of randomly selected stocks will have a standard deviation of returns that is greater than the standard deviation of a 1-stock portfolio if that one stock has a beta less than 1.0. c. A large portfolio of stocks whose betas are greater than 1.0 will have less market risk than a single stock with a beta = 0.8. d. If you add enough randomly selected stocks to a portfolio, you can completely eliminate all of the market risk from the portfolio. e. A large portfolio of randomly selected stocks will always have a standard deviation of returns that is less than the standard deviation of a portfolio with fewer stocks, regardless of how the stocks in the smaller portfolio are selected.

a. Diversifiable risk can be reduced by forming a large portfolio, but normally even highly-diversified portfolios are subject to market (or systematic) risk.

If markets are in equilibrium, which of the following conditions will exist? a. Each stock's expected return should equal its required return as seen by the marginal investor. b. All stocks should have the same expected return as seen by the marginal investor. c. The expected and required returns on stocks and bonds should be equal. d. All stocks should have the same realized return during the coming year. e. Each stock's expected return should equal its realized return as seen by the marginal investor.

a. Each stock's expected return should equal its required return as seen by the marginal investor.

Stock LB has a beta of 0.5 and Stock HB has a beta of 1.5. The market is in equilibrium, with required returns equaling expected returns. Which of the following statements is CORRECT? a. If both expected inflation and the market risk premium (rM − rRF) increase, the required return on Stock HB will increase by more than that on Stock LB. b. If both expected inflation and the market risk premium (rM − rRF) increase, the required returns of both stocks will increase by the same amount. c. Since the market is in equilibrium, the required returns of the two stocks should be the same. d. If expected inflation remains constant but the market risk premium (rM − rRF) declines, the required return of Stock HB will decline but the required return of Stock LB will increase. e. If expected inflation remains constant but the market risk premium (rM − rRF) declines, the required return of Stock LB will decline but the required return of Stock HB will increase.

a. If both expected inflation and the market risk premium (rM − rRF) increase, the required return on Stock HB will increase by more than that on Stock LB.

Stock A has a beta = 0.8, while Stock B has a beta = 1.6. Which of the following statements is CORRECT? a. If the marginal investor becomes more risk averse, the required return on Stock B will increase by more than the required return on Stock A. b. An equally weighted portfolio of Stocks A and B will have a beta lower than 1.2. c. If the marginal investor becomes more risk averse, the required return on Stock A will increase by more than the required return on Stock B. d. If the risk-free rate increases but the market risk premium remains constant, the required return on Stock A will increase by more than that on Stock B. e. Stock B's required return is double that of Stock A's.

a. If the marginal investor becomes more risk averse, the required return on Stock B will increase by more than the required return on Stock A.

Which of the following statements is CORRECT? a. Suppose the returns on two stocks are negatively correlated. One has a beta of 1.2 as determined in a regression analysis using data for the last 5 years, while the other has a beta of −0.6. The returns on the stock with the negative beta must have been negatively correlated with returns on most other stocks during that 5-year period. b. Suppose you are managing a stock portfolio, and you have information that leads you to believe the stock market is likely to be very strong in the immediate future. That is, you are convinced that the market is about to rise sharply. You should sell your high-beta stocks and buy low-beta stocks in order to take advantage of the expected market move. c. You think that investor sentiment is about to change, and investors are about to become more risk averse. This suggests that you should re-balance your portfolio to include more high-beta stocks. d. If the market risk premium remains constant, but the risk-free rate declines, then the required returns on low-beta stocks will rise while those on high-beta stocks will decline. e. Paid-in-Full Inc. is in the business of collecting past-due accounts for other companies, i.e., it is a collection agency. Paid-in-Full's revenues, profits, and stock price tend to rise during recessions. This suggests that Paid-in-Full Inc.'s beta should be quite high, say 2.0, because it does so much better than most other companies when the economy is weak.

a. Suppose the returns on two stocks are negatively correlated. One has a beta of 1.2 as determined in a regression analysis using data for the last 5 years, while the other has a beta of −0.6. The returns on the stock with the negative beta must have been negatively correlated with returns on most other stocks during that 5-year period.

Charlie and Lucinda each have $50,000 invested in stock portfolios. Charlie's has a beta of 1.2, an expected return of 10.8%, and a standard deviation of 25%. Lucinda's has a beta of 0.8, an expected return of 9.2%, and a standard deviation that is also 25%. The correlation coefficient, r, between Charlie's and Lucinda's portfolios is zero. If Charlie and Lucinda marry and combine their portfolios, which of the following best describes their combined $100,000 portfolio? a. The combined portfolio's beta will be equal to a simple weighted average of the betas of the two individual portfolios, 1.0; its expected return will be equal to a simple weighted average of the expected returns of the two individual portfolios, 10.0%; and its standard deviation will be less than the simple average of the two portfolios' standard deviations, 25%. b. The combined portfolio's expected return will be greater than the simple weighted average of the expected returns of the two individual portfolios, 10.0%. c. The combined portfolio's standard deviation will be greater than the simple average of the two portfolios' standard deviations, 25%. d. The combined portfolio's standard deviation will be equal to a simple average of the two portfolios' standard deviations, 25%. e. The combined portfolio's expected return will be less than the simple weighted average of the expected returns of the two individual portfolios, 10.0%.

a. The combined portfolio's beta will be equal to a simple weighted average of the betas of the two individual portfolios, 1.0; its expected return will be equal to a simple weighted average of the expected returns of the two individual portfolios, 10.0%; and its standard deviation will be less than the simple average of the two portfolios' standard deviations, 25%.

Stocks A and B each have an expected return of 15%, a standard deviation of 20%, and a beta of 1.2. The returns on the two stocks have a correlation coefficient of +0.6. Your portfolio consists of 50% A and 50% B. Which of the following statements is CORRECT? a. The portfolio's expected return is 15%. b. The portfolio's standard deviation is greater than 20%. c. The portfolio's beta is greater than 1.2. d. The portfolio's standard deviation is 20%. e. The portfolio's beta is less than 1.2.

a. The portfolio's expected return is 15%.

Portfolio P has equal amounts invested in each of the three stocks, A, B, and C. Stock A has a beta of 0.8, Stock B has a beta of 1.0, and Stock C has a beta of 1.2. Each of the stocks has a standard deviation of 25%. The returns on the three stocks are independent of one another (i.e., the correlation coefficients all equal zero). Assume that there is an increase in the market risk premium, but the risk-free rate remains unchanged. Which of the following statements is CORRECT? a. The required return on Stock A will increase by less than the increase in the market risk premium, while the required return on Stock C will increase by more than the increase in the market risk premium. b. The required return on the average stock will remain unchanged, but the returns of riskier stocks (such as Stock C) will increase while the returns of safer stocks (such as Stock A) will decrease. c. The required returns on all three stocks will increase by the amount of the increase in the market risk premium. d. The required return on the average stock will remain unchanged, but the returns on riskier stocks (such as Stock C) will decrease while the returns on safer stocks (such as Stock A) will increase. e. The required return of all stocks will remain unchanged since there was no change in their betas.

a. The required return on Stock A will increase by less than the increase in the market risk premium, while the required return on Stock C will increase by more than the increase in the market risk premium.

Assume that in recent years both expected inflation and the market risk premium (rM − rRF) have declined. Assume also that all stocks have positive betas. Which of the following would be most likely to have occurred as a result of these changes? a. The required returns on all stocks have fallen, but the fall has been greater for stocks with higher betas. b. The average required return on the market, rM, has remained constant, but the required returns have fallen for stocks that have betas greater than 1.0. c. Required returns have increased for stocks with betas greater than 1.0 but have declined for stocks with betas less than 1.0. d. The required returns on all stocks have fallen by the same amount. e. The required returns on all stocks have fallen, but the decline has been greater for stocks with lower betas.

a. The required returns on all stocks have fallen, but the fall has been greater for stocks with higher betas.

Stock A has an expected return of 12%, a beta of 1.2, and a standard deviation of 20%. Stock B also has a beta of 1.2, but its expected return is 10% and its standard deviation is 15%. Portfolio AB has $300,000 invested in Stock A and $100,000 invested in Stock B. The correlation between the two stocks' returns is zero (that is, rA,B = 0). Which of the following statements is CORRECT? a. The stocks are not in equilibrium based on the CAPM; if A is valued correctly, then B is overvalued. b. The stocks are not in equilibrium based on the CAPM; if A is valued correctly, then B is undervalued. c. Portfolio AB's expected return is 11.0%. d. Portfolio AB's beta is less than 1.2. e. Portfolio AB's standard deviation is 17.5%.

a. The stocks are not in equilibrium based on the CAPM; if A is valued correctly, then B is overvalued.

You are considering investing in one of the these three stocks: Stock Standard Deviation Beta A 20% 0.59 B 10% 0.61 C 12% 1.29 If you are a strict risk minimizer, you would choose Stock ____ if it is to be held in isolation and Stock ____ if it is to be held as part of a well-diversified portfolio. a. A; B. b. B; A. c. C; A. d. C; B. e. A; A.

b. B; A.

The risk-free rate is 6%; Stock A has a beta of 1.0; Stock B has a beta of 2.0; and the market risk premium, rM − rRF, is positive. Which of the following statements is CORRECT? a. Stock B's required rate of return is twice that of Stock A. b. If Stock A's required return is 11%, then the market risk premium is 5%. c. If Stock B's required return is 11%, then the market risk premium is 5%. d. If the risk-free rate remains constant but the market risk premium increases, Stock A's required return will increase by more than Stock B's. e. If the risk-free rate increases but the market risk premium stays unchanged, Stock B's required return will increase by more than Stock A's.

b. If Stock A's required return is 11%, then the market risk premium is 5%.

Stock X has a beta of 0.7 and Stock Y has a beta of 1.7. Which of the following statements must be true, according to the CAPM? a. Stock Y's realized return during the coming year will be higher than Stock X's return. b. If the expected rate of inflation increases but the market risk premium is unchanged, the required returns on the two stocks should increase by the same amount. c. Stock Y's return has a higher standard deviation than Stock X. d. If the market risk premium declines, but the risk-free rate is unchanged, Stock X will have a larger decline in its required return than will Stock Y. e. If you invest $50,000 in Stock X and $50,000 in Stock Y, your 2-stock portfolio would have a beta significantly lower than 1.0, provided the returns on the two stocks are not perfectly correlated.

b. If the expected rate of inflation increases but the market risk premium is unchanged, the required returns on the two stocks should increase by the same amount.

Which of the following statements is CORRECT? a. If you were restricted to investing in publicly traded common stocks, yet you wanted to minimize the riskiness of your portfolio as measured by its beta, then according to the CAPM theory you should invest an equal amount of money in each stock in the market. That is, if there were 10,000 traded stocks in the world, the least risky possible portfolio would include some shares of each one. b. If you formed a portfolio that consisted of all stocks with betas less than 1.0, which is about half of all stocks, the portfolio would itself have a beta coefficient that is equal to the weighted average beta of the stocks in the portfolio, and that portfolio would have less risk than a portfolio that consisted of all stocks in the market. c. Market risk can be eliminated by forming a large portfolio, and if some Treasury bonds are held in the portfolio, the portfolio can be made to be completely riskless. d. A portfolio that consists of all stocks in the market would have a required return that is equal to the riskless rate. e. If you add enough randomly selected stocks to a portfolio, you can completely eliminate all of the market risk from the portfolio.

b. If you formed a portfolio that consisted of all stocks with betas less than 1.0, which is about half of all stocks, the portfolio would itself have a beta coefficient that is equal to the weighted average beta of the stocks in the portfolio, and that portfolio would have less risk than a portfolio that consisted of all stocks in the market.

In historical data, we see that investments with the highest average annual returns also tend to have the highest standard deviations of annual returns. This observation supports the notion that there is a positive correlation between risk and return. Which of the following answers correctly ranks investments from highest to lowest risk (and return), where the security with the highest risk is shown first, the one with the lowest risk last? a. Large-company stocks, small-company stocks, long-term corporate bonds, U.S. Treasury bills, long-term government bonds. b. Small-company stocks, large-company stocks, long-term corporate bonds, long-term government bonds, U.S. Treasury bills. c. U.S. Treasury bills, long-term government bonds, long-term corporate bonds, small-company stocks, large-company stocks. d. Large-company stocks, small-company stocks, long-term corporate bonds, long-term government bonds, U.S. Treasury bills. e. Small-company stocks, long-term corporate bonds, large-company stocks, long-term government bonds, U.S. Treasury bills.

b. Small-company stocks, large-company stocks, long-term corporate bonds, long-term government bonds, U.S. Treasury bills.

Which of the following statements is CORRECT? a. The SML shows the relationship between companies' required returns and their diversifiable risks. The slope and intercept of this line cannot be influenced by a firm's managers, but the position of the company on the line can be influenced by its managers. b. Suppose you plotted the returns of a given stock against those of the market, and you found that the slope of the regression line was negative. The CAPM would indicate that the required rate of return on the stock should be less than the risk-free rate for a well-diversified investor, assuming investors expect the observed relationship to continue on into the future. c. If investors become less risk averse, the slope of the Security Market Line will increase. d. If a company increases its use of debt, this is likely to cause the slope of its SML to increase, indicating a higher required return on the stock. e. The slope of the SML is determined by the value of beta.

b. Suppose you plotted the returns of a given stock against those of the market, and you found that the slope of the regression line was negative. The CAPM would indicate that the required rate of return on the stock should be less than the risk-free rate for a well-diversified investor, assuming investors expect the observed relationship to continue on into the future.

Which of the following statements is CORRECT? a. If you found a stock with a zero historical beta and held it as the only stock in your portfolio, you would by definition have a riskless portfolio. b. The beta coefficient of a stock is normally found by regressing past returns on a stock against past market returns. One could also construct a scatter diagram of returns on the stock versus those on the market, estimate the slope of the line of best fit, and use it as beta. However, this historical beta may differ from the beta that exists in the future. c. The beta of a portfolio of stocks is always larger than the betas of any of the individual stocks. d. It is theoretically possible for a stock to have a beta of 1.0. If a stock did have a beta of 1.0, then, at least in theory, its required rate of return would be equal to the risk-free (default-free) rate of return, rRF. e. The beta of a portfolio of stocks is always smaller than the betas of any of the individual stocks.

b. The beta coefficient of a stock is normally found by regressing past returns on a stock against past market returns. One could also construct a scatter diagram of returns on the stock versus those on the market, estimate the slope of the line of best fit, and use it as beta. However, this historical beta may differ from the beta that exists in the future.

Which of the following is NOT a potential problem when estimating and using betas, i.e., which statement is FALSE? a. Sometimes, during a period when the company is undergoing a change such as toward more leverage or riskier assets, the calculated beta will be drastically different from the "true" or "expected future" beta. b. The beta of an "average stock," or "the market," can change over time, sometimes drastically. c. Sometimes the past data used to calculate beta do not reflect the likely risk of the firm for the future because conditions have changed. d. All of the statements above are true. e. The fact that a security or project may not have a past history that can be used as the basis for calculating beta.

b. The beta of an "average stock," or "the market," can change over time, sometimes drastically.

In a portfolio of three randomly selected stocks, which of the following could NOT be true; i.e., which statement is false? a. The standard deviation of the portfolio is greater than the standard deviation of one or two of the stocks. b. The beta of the portfolio is lower than the lowest of the three betas. c. The beta of the portfolio is equal to one of the three stock's betas. d. The beta of the portfolio is equal to 1. e. The standard deviation of the portfolio is less than the standard deviation of each of the stocks if they were held in isolation.

b. The beta of the portfolio is lower than the lowest of the three betas.

. Suppose that during the coming year, the risk free rate, rRF, is expected to remain the same, while the market risk premium (rM − rRF), is expected to fall. Given this forecast, which of the following statements is CORRECT? a. The required return on all stocks will remain unchanged. b. The required return will fall for all stocks, but it will fall more for stocks with higher betas. c. The required return for all stocks will fall by the same amount. d. The required return will fall for all stocks, but it will fall less for stocks with higher betas. e. The required return will increase for stocks with a beta less than 1.0 and will decrease for stocks with a beta greater than 1.0.

b. The required return will fall for all stocks, but it will fall more for stocks with higher betas.

Ann has a portfolio of 20 average stocks, and Tom has a portfolio of 2 average stocks. Assuming the market is in equilibrium, which of the following statements is CORRECT? a. The required return on Ann's portfolio will be lower than that on Tom's portfolio because Ann's portfolio will have less total risk. b. Tom's portfolio will have more diversifiable risk, the same market risk, and thus more total risk than Ann's portfolio, but the required (and expected) returns will be the same on both portfolios. c. If the two portfolios have the same beta, their required returns will be the same, but Ann's portfolio will have less market risk than Tom's. d. The expected return on Jane's portfolio must be lower than the expected return on Dick's portfolio because Jane is more diversified. e. Ann's portfolio will have less diversifiable risk and also less market risk than Tom's portfolio.

b. Tom's portfolio will have more diversifiable risk, the same market risk, and thus more total risk than Ann's portfolio, but the required (and expected) returns will be the same on both portfolios

The two stocks in your portfolio, X and Y, have independent returns, so the correlation between them, rXY is zero. Your portfolio consists of $50,000 invested in Stock X and $50,000 invested in Stock Y. Both stocks have an expected return of 15%, betas of 1.6, and standard deviations of 30%. Which of the following statements best describes the characteristics of your 2-stock portfolio? a. Your portfolio has a standard deviation less than 30%, and its beta is greater than 1.6. b. Your portfolio has a beta equal to 1.6, and its expected return is 15%. c. Your portfolio has a beta greater than 1.6, and its expected return is greater than 15%. d. Your portfolio has a standard deviation greater than 30% and a beta equal to 1.6. e. Your portfolio has a standard deviation of 30%, and its expected return is 15%.

b. Your portfolio has a beta equal to 1.6, and its expected return is 15%.

Recession, inflation, and high interest rates are economic events that are best characterized as being a. company-specific risk factors that can be diversified away. b. among the factors that are responsible for market risk. c. risks that are beyond the control of investors and thus should not be considered by security analysts or portfolio managers. d. irrelevant except to governmental authorities like the Federal Reserve. e. systematic risk factors that can be diversified away.

b. among the factors that are responsible for market risk.

Stock X has a beta of 0.6, while Stock Y has a beta of 1.4. Which of the following statements is CORRECT? a. Stock Y must have a higher expected return and a higher standard deviation than Stock X. b. If expected inflation increases but the market risk premium is unchanged, then the required return on both stocks will fall by the same amount. c. If the market risk premium declines but expected inflation is unchanged, the required return on both stocks will decrease, but the decrease will be greater for Stock Y. d. If expected inflation declines but the market risk premium is unchanged, then the required return on both stocks will decrease but the decrease will be greater for Stock Y. e. A portfolio consisting of $50,000 invested in Stock X and $50,000 invested in Stock Y will have a required return that exceeds that of the overall market.

c. If the market risk premium declines but expected inflation is unchanged, the required return on both stocks will decrease, but the decrease will be greater for Stock Y.

Which of the following statements is CORRECT? (Assume that the risk-free rate is a constant.) a. The effect of a change in the market risk premium depends on the slope of the yield curve. b. If the market risk premium increases by 1%, then the required return on all stocks will rise by 1%. c. If the market risk premium increases by 1%, then the required return will increase by 1% for a stock that has a beta of 1.0. d. The effect of a change in the market risk premium depends on the level of the risk-free rate. e. If the market risk premium increases by 1%, then the required return will increase for stocks that have a beta greater than 1.0, but it will decrease for stocks that have a beta less than 1.0.

c. If the market risk premium increases by 1%, then the required return will increase by 1% for a stock that has a beta of 1.0.

Dixon Food's stock has a beta of 1.4, while Clark Café's stock has a beta of 0.7. Assume that the risk-free rate, rRF, is 5.5% and the market risk premium, (rM − rRF), equals 4%. Which of the following statements is CORRECT? a. If the market risk premium increases but the risk-free rate remains unchanged, Dixon's required return will increase because it has a beta greater than 1.0 but Clark's required return will decline because it has a beta less than 1.0. b. Since Dixon's beta is twice that of Clark's, its required rate of return will also be twice that of Clark's. c. If the risk-free rate increases while the market risk premium remains constant, then the required return on an average stock will increase. d. If the market risk premium decreases but the risk-free rate remains unchanged, Dixon's required return will decrease because it has a beta greater than 1.0 and Clark's will also decrease, but by more than Dixon's because it has a beta less than 1.0. e. If the risk-free rate increases but the market risk premium remains unchanged, the required return will increase for both stocks but the increase will be larger for Dixon since it has a higher beta.

c. If the risk-free rate increases while the market risk premium remains constant, then the required return on an average stock will increase.

Consider the following information for three stocks, A, B, and C. The stocks' returns are positively but not perfectly positively correlated with one another, i.e., the correlations are all between 0 and 1. Expected Standard Stock Return Deviation Beta A 10% 20% 1.0 B 10% 10% 1.0 C 12% 12% 1.4 Portfolio AB has half of its funds invested in Stock A and half in Stock B. Portfolio ABC has one third of its funds invested in each of the three stocks. The risk-free rate is 5%, and the market is in equilibrium, so required returns equal expected returns. Which of the following statements is CORRECT? a. Portfolio AB's coefficient of variation is greater than 2.0. b. Portfolio AB's required return is greater than the required return on Stock A. c. Portfolio ABC's expected return is 10.66667%. d. Portfolio ABC has a standard deviation of 20%. e. Portfolio AB has a standard deviation of 20%.

c. Portfolio ABC's expected return is 10.66667%.

Stocks A and B are quite similar: Each has an expected return of 12%, a beta of 1.2, and a standard deviation of 25%. The returns on the two stocks have a correlation of 0.6. Portfolio P has 50% in Stock A and 50% in Stock B. Which of the following statements is CORRECT? a. Portfolio P has a standard deviation that is greater than 25%. b. Portfolio P has an expected return that is less than 12%. c. Portfolio P has a standard deviation that is less than 25%. d. Portfolio P has a beta that is less than 1.2. e. Portfolio P has a beta that is greater than 1.2.

c. Portfolio P has a standard deviation that is less than 25%.

Stock A has a beta of 0.8, Stock B has a beta of 1.0, and Stock C has a beta of 1.2. Portfolio P has 1/3 of its value invested in each stock. Each stock has a standard deviation of 25%, and their returns are independent of one another, i.e., the correlation coefficients between each pair of stocks is zero. Assuming the market is in equilibrium, which of the following statements is CORRECT? a. Portfolio P's expected return is equal to the expected return on Stock A. b. Portfolio P's expected return is less than the expected return on Stock B. c. Portfolio P's expected return is equal to the expected return on Stock B. d. Portfolio P's expected return is greater than the expected return on Stock C. e. Portfolio P's expected return is greater than the expected return on Stock B.

c. Portfolio P's expected return is equal to the expected return on Stock B

Which of the following is most likely to be true for a portfolio of 40 randomly selected stocks? a. The riskiness of the portfolio is the same as the riskiness of each stock if it was held in isolation. b. The beta of the portfolio is less than the average of the betas of the individual stocks. c. The beta of the portfolio is equal to the average of the betas of the individual stocks. d. The beta of the portfolio is larger than the average of the betas of the individual stocks. e. The riskiness of the portfolio is greater than the riskiness of each of the stocks if each was held in isolation.

c. The beta of the portfolio is equal to the average of the betas of the individual stocks.

Stock A's beta is 1.7 and Stock B's beta is 0.7. Which of the following statements must be true about these securities? (Assume market equilibrium.) a. Stock B must be a more desirable addition to a portfolio than A. b. Stock A must be a more desirable addition to a portfolio than B. c. The expected return on Stock A should be greater than that on B. d. The expected return on Stock B should be greater than that on A. e. When held in isolation, Stock A has more risk than Stock B.

c. The expected return on Stock A should be greater than that on B

Consider the following average annual returns for Stocks A and B and the Market. Which of the possible answers best describes the historical betas for A and B? Years Market Stock A Stock B 1 0.03 0.16 0.05 2 −0.05 0.20 0.05 3 0.01 0.18 0.05 4 −0.10 0.25 0.05 5 0.06 0.14 0.05 a. bA > +1; bB = 0. b. bA = 0; bB = −1. c. bA < 0; bB = 0. d. bA < −1; bB = 1. e. bA > 0; bB = 1.

c. bA < 0; bB = 0. (First, note that B's beta must be zero, so either a or c must be correct. Second, note that A's returns are highest when the market's returns are negative and lowest when the market's returns are positive. This indicates that A's beta is negative. Thus, c must be correct.)

Which of the following statements is CORRECT? a. If an investor buys enough stocks, he or she can, through diversification, eliminate all of the diversifiable risk inherent in owning stocks. Therefore, if a portfolio contained all publicly traded stocks, it would be essentially riskless. b. The required return on a firm's common stock is, in theory, determined solely by its market risk. If the market risk is known, and if that risk is expected to remain constant, then no other information is required to specify the firm's required return. c. Portfolio diversification reduces the variability of returns (as measured by the standard deviation) of each individual stock held in a portfolio. d. A security's beta measures its non-diversifiable, or market, risk relative to that of an average stock. e. A stock's beta is less relevant as a measure of risk to an investor with a well-diversified portfolio than to an investor who holds only that one stock.

d. A security's beta measures its non-diversifiable, or market, risk relative to that of an average stock.

Which of the following statements is CORRECT? a. The higher the correlation between the stocks in a portfolio, the lower the risk inherent in the portfolio. b. An investor can eliminate almost all risk if he or she holds a very large and well diversified portfolio of stocks. c. Once a portfolio has about 40 stocks, adding additional stocks will not reduce its risk by even a small amount. d. An investor can eliminate almost all diversifiable risk if he or she holds a very large, well-diversified portfolio of stocks. e. An investor can eliminate almost all market risk if he or she holds a very large and well diversified portfolio of stocks.

d. An investor can eliminate almost all diversifiable risk if he or she holds a very large, well-diversified portfolio of stocks.

Which of the following statements is CORRECT? a. Logically, it is easier to estimate the betas associated with capital budgeting projects than the betas associated with stocks, especially if the projects are closely associated with research and development activities. b. The beta of an "average stock," which is also "the market beta," can change over time, sometimes drastically. c. If a newly issued stock does not have a past history that can be used for calculating beta, then we should always estimate that its beta will turn out to be 1.0. This is especially true if the company finances with more debt than the average firm. d. During a period when a company is undergoing a change such as increasing its use of leverage or taking on riskier projects, the calculated historical beta may be drastically different from the beta that will exist in the future. e. If a company with a high beta merges with a low-beta company, the best estimate of the new merged company's beta is 1.0.

d. During a period when a company is undergoing a change such as increasing its use of leverage or taking on riskier projects, the calculated historical beta may be drastically different from the beta that will exist in the future.

Which of the following statements is CORRECT? a. Other things held constant, if investors suddenly become convinced that there will be deflation in the economy, then the required returns on all stocks should increase. b. If a company's beta were cut in half, then its required rate of return would also be halved. c. If the risk-free rate rises by 0.5% but the market risk premium declines by that same amount, then the required rates of return on stocks with betas less than 1.0 will decline while returns on stocks with betas above 1.0 will increase. d. If the risk-free rate rises by 0.5% but the market risk premium declines by that same amount, then the required rate of return on an average stock will remain unchanged, but required returns on stocks with betas less than 1.0 will rise. e. If a company's beta doubles, then its required rate of return will also double.

d. If the risk-free rate rises by 0.5% but the market risk premium declines by that same amount, then the required rate of return on an average stock will remain unchanged, but required returns on stocks with betas less than 1.0 will rise.

Stock A's beta is 1.7 and Stock B's beta is 0.7. Which of the following statements must be true, assuming the CAPM is correct. a. In equilibrium, the expected return on Stock B will be greater than that on Stock A. b. When held in isolation, Stock A has more risk than Stock B. c. Stock B would be a more desirable addition to a portfolio than A. d. In equilibrium, the expected return on Stock A will be greater than that on B. e. Stock A would be a more desirable addition to a portfolio then Stock B.

d. In equilibrium, the expected return on Stock A will be greater than that on B.

Stocks A, B, and C are similar in some respects: Each has an expected return of 10% and a standard deviation of 25%. Stocks A and B have returns that are independent of one another; i.e., their correlation coefficient, r, equals zero. Stocks A and C have returns that are negatively correlated with one another; i.e., r is less than 0. Portfolio AB is a portfolio with half of its money invested in Stock A and half in Stock B. Portfolio AC is a portfolio with half of its money invested in Stock A and half invested in Stock C. Which of the following statements is CORRECT? a. Portfolio AC has an expected return that is greater than 25%. b. Portfolio AB has a standard deviation that is greater than 25%. c. Portfolio AB has a standard deviation that is equal to 25%. d. Portfolio AC has a standard deviation that is less than 25%. e. Portfolio AC has an expected return that is less than 10%.

d. Portfolio AC has a standard deviation that is less than 25%.

Portfolio P has $200,000 consisting of $100,000 invested in Stock A and $100,000 in Stock B. Stock A has a beta of 1.2 and a standard deviation of 20%. Stock B has a beta of 0.8 and a standard deviation of 25%. Which of the following statements is CORRECT? (Assume that the stocks are in equilibrium.) a. Stock B has a higher required rate of return than Stock A. b. Portfolio P has a standard deviation of 22.5%. c. More information is needed to determine the portfolio's beta. d. Portfolio P has a beta of 1.0. e. Stock A's returns are less highly correlated with the returns on most other stocks than are B's returns.

d. Portfolio P has a beta of 1.0.

You have a portfolio P that consists of 50% Stock X and 50% Stock Y. Stock X has a beta of 0.7 and Stock Y has a beta of 1.3. The standard deviation of each stock's returns is 20%. The stocks' returns are independent of each other, i.e., the correlation coefficient, r, between them is zero. Given this information, which of the following statements is CORRECT? a. The required return on Portfolio P is equal to the market risk premium (rM − rRF). b. Portfolio P has a beta of 0.7. c. Portfolio P has a beta of 1.0 and a required return that is equal to the riskless rate, rRF. d. Portfolio P has the same required return as the market (rM). e. Portfolio P has a standard deviation of 20%.

d. Portfolio P has the same required return as the market (rM).

Assume that the risk-free rate remains constant, but the market risk premium declines. Which of the following is most likely to occur? a. The required return on a stock with beta > 1.0 will increase. b. The return on "the market" will remain constant. c. The return on "the market" will increase. d. The required return on a stock with beta < 1.0 will decline. e. The required return on a stock with beta = 1.0 will not change.

d. The required return on a stock with beta < 1.0 will decline.

Which of the following statements is CORRECT? a. If the risk-free rate rises, then the market risk premium must also rise. b. If a company's beta is halved, then its required return will also be halved. c. If a company's beta doubles, then its required return will also double. d. The slope of the security market line is equal to the market risk premium, (rM − rRF). e. Beta is measured by the slope of the security market line.

d. The slope of the security market line is equal to the market risk premium, (rM − rRF).

If you randomly select stocks and add them to your portfolio, which of the following statements best describes what you should expect? a. Adding more such stocks will increase the portfolio's expected rate of return. b. Adding more such stocks will reduce the portfolio's beta coefficient and thus its systematic risk. c. Adding more such stocks will have no effect on the portfolio's risk. d. Adding more such stocks will reduce the portfolio's market risk but not its unsystematic risk. e. Adding more such stocks will reduce the portfolio's unsystematic, or diversifiable, risk.

e. Adding more such stocks will reduce the portfolio's unsystematic, or diversifiable, risk

Assume that the risk-free rate is 6% and the market risk premium is 5%. Given this information, which of the following statements is CORRECT? a. If a stock has a negative beta, its required return must also be negative. b. An index fund with beta = 1.0 should have a required return less than 11%. c. If a stock's beta doubles, its required return must also double. d. An index fund with beta = 1.0 should have a required return greater than 11%. e. An index fund with beta = 1.0 should have a required return of 11%.

e. An index fund with beta = 1.0 should have a required return of 11%.

Assume that the risk-free rate is 5%. Which of the following statements is CORRECT? a. If a stock's beta doubled, its required return under the CAPM would also double. b. If a stock's beta doubled, its required return under the CAPM would more than double. c. If a stock's beta were 1.0, its required return under the CAPM would be 5%. d. If a stock's beta were less than 1.0, its required return under the CAPM would be less than 5%. e. If a stock has a negative beta, its required return under the CAPM would be less than 5%.

e. If a stock has a negative beta, its required return under the CAPM would be less than 5%.

Which of the following is most likely to occur as you add randomly selected stocks to your portfolio, which currently consists of 3 average stocks? a. The expected return of your portfolio is likely to decline. b. The diversifiable risk will remain the same, but the market risk will likely decline. c. Both the diversifiable risk and the market risk of your portfolio are likely to decline. d. The total risk of your portfolio should decline, and as a result, the expected rate of return on the portfolio should also decline. e. The diversifiable risk of your portfolio will likely decline, but the expected market risk should not change.

e. The diversifiable risk of your portfolio will likely decline, but the expected market risk should not change.

Stock A has a beta of 0.7, whereas Stock B has a beta of 1.3. Portfolio P has 50% invested in both A and B. Which of the following would occur if the market risk premium increased by 1% but the risk-free rate remained constant? a. The required return on both stocks would increase by 1%. b. The required return on Portfolio P would remain unchanged. c. The required return on Stock A would increase by more than 1%, while the return on Stock B would increase by less than 1%. d. The required return for Stock A would fall, but the required return for Stock B would increase. e. The required return on Portfolio P would increase by 1%.

e. The required return on Portfolio P would increase by 1%.

98. Stock A has a beta of 0.8 and Stock B has a beta of 1.2. 50% of Portfolio P is invested in Stock A and 50% is invested in Stock B. If the market risk premium (rM − rRF) were to increase but the risk-free rate (rRF) remained constant, which of the following would occur? a. The required return would decrease by the same amount for both Stock A and Stock B. b. The required return would increase for Stock A but decrease for Stock B. c. The required return on Portfolio P would remain unchanged. d. The required return would increase for Stock B but decrease for Stock A. e. The required return would increase for both stocks but the increase would be greater for Stock B than for Stock A.

e. The required return would increase for both stocks but the increase would be greater for Stock B than for Stock A.

Which of the following statements is CORRECT? a. Lower beta stocks have higher required returns. b. A stock's beta indicates its diversifiable risk. c. Diversifiable risk cannot be completely diversified away. d. Two securities with the same stand-alone risk must have the same betas. e. The slope of the security market line is equal to the market risk premium.

e. The slope of the security market line is equal to the market risk premium

How would the Security Market Line be affected, other things held constant, if the expected inflation rate decreases and investors also become more risk averse? a. The x-axis intercept would decline, and the slope would increase. b. The y-axis intercept would increase, and the slope would decline. c. The SML would be affected only if betas changed. d. Both the y-axis intercept and the slope would increase, leading to higher required returns. e. The y-axis intercept would decline, and the slope would increase.

e. The y-axis intercept would decline, and the slope would increase


Set pelajaran terkait

Possible Test 3 Questions (Read Description)

View Set

Chapter 44: Digestive and Gastrointestinal Treatment Modalities

View Set

Quality Assurance and Radiology Regulations Quality Assurance

View Set

Introduction to Kinesiology, Chapter 5

View Set

Venezuela cultural questions Exam 1

View Set

Napoleon's Buttons Chapter 15: Salt

View Set